Use the drawing tool(s) to form the correct answer on the provided graph. Plot the axis of symmetry and the vertex for this function: h(x) = (x − 5)2 − 7.

Use The Drawing Tool(s) To Form The Correct Answer On The Provided Graph. Plot The Axis Of Symmetry And

Answers

Answer 1
vertex:
(h, k) = (5, -7)
the axis of symmetry is (5,0)

Related Questions

Can someone help me find the elevation of the sun I need the answers that are highlighted in yellow please help image below

Answers

Answer:

Step-by-step explanation:

a.  ∠ACB

b.  AC

c.  AB

d.  BC

e.  tangent, opposite, adjacent

f.  m∠ACB = tan⁻¹(34/45) = 37°

let be the linear transformation given by let be the basis of given by and let be the basis of given by find the coordinate matrix of relative to the ordered bases and . HW6.7. Coordinate matrix for differentiation Let L :P2P be the linear transformation given by L(p(t)) = 5p"(t) + 1p' (t) + 3p(t) + 3tp(t). Let E = (C1, C2, C3) be the basis of P2 given by el(t) = 1, ez(t) = t, ez(t) = ť. and let F = (f1, 82, 83, fa) be the basis of P3 given by fi(t) = 1, fz(t) = t, fz(t) = {2, fa(t) = {'. Find the coordinate matrix LFE of L relative to the ordered bases & and F. LFE = Save & Grade 2 tries left Save only

Answers

The coordinate matrix LFE of L relative to the ordered bases E and F is

[tex]\left(\begin{array}{ccc}3&1&10\\3&3&2\\0&3&3\\0&0&3\end{array}\right)[/tex].

Since here L is a linear transformation from a vector space of dimension 3 to a vector space of dimension 4, the coordinate matrix of L relative to the given ordered bases must be a (4×3) matrix,

Linear transformation is given by,

L(P(t)) = 5p"(t) + 1p' (t) + 3p(t) + 3tp(t)

The given basis is IP² is E = (C1, C2, C3) where, e1(t) = 1, e2(t) = t, e3(t) = t².

Also the given basis of IP³ is (f1, f2, f3, f4) where, f1(t) = 1, f2(t) = t, f3(t) = t², f4(t) = t³.

Now to find the coordinate matrix,

Now,

L(e1(t)) = 5.0 + 1.0 + 3.1 + 3t.1

= 3 + 3t

= 3f1(t) + 3f2(t) + 0.f3(t) + 0.f4(t)

L(e2(t)) = 5.0 + 1.1 + 3.t + 3t.t

= 1 + 3t + 3t²

L(e3(t)) = 5.2 + 1.2t + 3.t² + 3t.t²

= 10 + 2t + 3t² + 3t³

Now writing the coefficients as a column vector we get,

[tex]\left(\begin{array}{ccc}3&1&10\\3&3&2\\0&3&3\\0&0&3\end{array}\right)[/tex]

The coordinate matrix LFE of L relative to the ordered bases E and F is

[tex]\left(\begin{array}{ccc}3&1&10\\3&3&2\\0&3&3\\0&0&3\end{array}\right)[/tex].

Learn more about Coordinate matrix:

https://brainly.com/question/28194667

#SPJ4

Express the following as the product of prime factors in exponential form
(a) 432 (b) 729×64

Answers

Answer: 729×64 is: (3^3 × 2^3)^2

Step-by-step explanation:

(a) To express 432 as the product of prime factors in exponential form, we can follow these steps:

Divide by 2 as many times as possible until the result is odd: 432 ÷ 2 = 216 ÷ 2 = 108 ÷ 2 = 54 ÷ 2 = 27 (5 times)

Divide by 3 as many times as possible until the result is not divisible by 3: 27 ÷ 3 = 9 ÷ 3 = 3 (2 times)

Since 3 is a prime number, we cannot divide by any other prime number to obtain a smaller result. Therefore, the prime factorization of 432 is: 2^4 × 3^3.

(b) To express 729×64 as the product of prime factors in exponential form, we can follow these steps:

Rewrite each factor as a power of a prime: 729 = 3^6 and 64 = 2^6.

Multiply the powers of each prime together: (3^6) × (2^6) = 3^6 × 2^6.

Simplify the result by factoring out the highest possible power of each prime: 3^6 × 2^6 = (3^3 × 2^3)^2.

Therefore, the prime factorization of 729×64 is: (3^3 × 2^3)^2.

Answer:

Below in bold.

Step-by-step explanation:

2) 432

2) 216

2) 108

2)  54

3) 27

3) 9

   3

So 432 = 2^4 * 3^3.

3)729

3)243

3)81

3)27

3)9

3

64 = 2^6

So the answer is 2^6 * 3^6

What is the value of v i need help

Answers

Therefore , the solution of the given problem of unitary method comes out to be V has a value of -13/3.

What is a unitary method?

By using preexisting variables, this common convenience, or all essential components from the initial Bishop malleable study that adhered to a specific methodology, the objective may be accomplished. If the term affirmation result does not occur, both essential components will event miss the statement; however, if it does, it then becomes possible to reach the entity once more.

Here,

TU = 4v and QR = 5v plus 17 allow us to write:

=> QR = TU + UP + PS + SR + RQ

Inputting the numbers provided yields:

=> 5v + 17 =  4v + UP + v + 26 + SR + 5v plus

When we simplify the solution, we obtain:

=> UP + SR + 6v + 26 = 0

Because UP and SR are opposite and equivalent, we can write:

=> UP = -SR

By replacing this in the preceding solution, we obtain:

=> UP - UP + 6v + 26 = 0

When we simplify the solution, we obtain:

=> 6v + 26 = 0

26 is subtracted from both parts to yield:

=> 6v = -26

When we multiply both parts by 6, we get:

=> v = -26/6

If we simplify, we get:

=> v = -13/3

V therefore has a value of -13/3.

To know more about unitary method visit:

https://brainly.com/question/28276953

#SPJ1

At the book store, you purchased some $5 clearance mystery books and $12 regular-priced science fiction books. How many of each did you buy if you spent a total of $126?

Answers

Answer: View answer in explanation below.

Step-by-step explanation: Let's use variables to represent the unknown quantities.

Let x be the number of $5 clearance mystery books purchased.

Let y be the number of $12 regular-priced science fiction books purchased.

We can set up a system of equations based on the given information:

5x + 12y = 126 (total amount spent)

x + y = total number of books purchased

We need to solve for x and y.

Let's use the second equation to solve for one variable in terms of the other:

y = total number of books purchased - x

Now we can substitute this expression for y into the first equation:

5x + 12(total number of books purchased - x) = 126

Simplifying and solving for x:

5x + 12total number of books purchased - 12x = 126

-7x + 12total number of books purchased = 126

-7x = -12total number of books purchased + 126

x = (12total number of books purchased - 126)/7

Since x must be a whole number (you can't buy a fraction of a book), we need to find a value of total number of books purchased that makes x a whole number. We can start by trying different values of total number of books purchased:

If total number of books purchased is 10:

x = (12(10) - 126)/7 = -6/7 (not a whole number)

If total number of books purchased is 11:

x = (12(11) - 126)/7 = 6/7 (not a whole number)

If total number of books purchased is 12:

x = (12(12) - 126)/7 = 6/7 (not a whole number)

If total number of books purchased is 13:

x = (12(13) - 126)/7 = 12/7 (not a whole number)

If total number of books purchased is 14:

x = (12(14) - 126)/7 = 18/7 (not a whole number)

If total number of books purchased is 15:

x = (12(15) - 126)/7 = 24/7 (not a whole number)

If total number of books purchased is 16:

x = (12(16) - 126)/7 = 30/7 (not a whole number)

If total number of books purchased is 17:

x = (12(17) - 126)/7 = 36/7 (not a whole number)

If total number of books purchased is 18:

x = (12(18) - 126)/7 = 42/7 = 6 (a whole number)

So, you bought 6 $5 clearance mystery books and 12 - 6 = 6 $12 regular-priced science fiction books.

Daniel is paid an hourly rate of $9. 00 plus six percent commission on direct phone sales. Last week he worked 48 hours and received $72. 00 in commissions. What was his pay for the week?

Answers

Daniel's pay for the week was $504.00, which includes his hourly pay and his commission on direct phone sales.

To calculate Daniel's pay for the week, we need to find his total earnings, which include his hourly wage and his commission on direct phone sales.

First, let's calculate his commission on direct phone sales. We know that he received $72.00 in commissions, and we also know that his commission is six percent of his sales. So, we can use the formula:

Commission = Sales x Commission Rate

To find his sales, we can rearrange the formula to:

Sales = Commission / Commission Rate

Plugging in the given values, we get:

Sales = $72.00 / 0.06 = $1200.00

So, Daniel's direct phone sales for the week were $1200.00.

Now, let's calculate his hourly pay. He worked for 48 hours at a rate of $9.00 per hour, so his hourly earnings were:

Hourly Pay = Hours Worked x Hourly Rate

Hourly Pay = 48 x $9.00 = $432.00

Finally, we can add his commission and his hourly pay to find his total earnings for the week:

Total Pay = Hourly Pay + Commission

Total Pay = $432.00 + $72.00

Total Pay = $504.00

To learn more about pay click on,

https://brainly.com/question/3473007

#SPJ4


[tex]f(x) = \frac{ - 3}{x - 2} + 1[/tex]
Sketch the graph of f clearly showing the asymptotes and the intercepts with the axis​

Answers

Answer:  Vertical asymptote = 2

                Horizontal asymptote = 1

                y intercept = ( 0, 5/2 )

                x intercept = ( 5,0 )

Other points of graph = ( 3,-2 )

                                      = ( 4,-1/2 )

                                      = ( -1, 2 )

                                      = (-2,7/4 )

                                      = ( -3 , 8/5 )

Step-by-step explanation:  

A right triangle is describe as having an angle of measure six less than negative two times a number, another angle measure that is three less than negative one-fourth the number, and a right angle. What are the measure of the angles in degree

Answers

The angles measure 90°, -2x - 6, and -1/x - 3. ⇒ x = -44. Therefore, the required measures of the angles are 90°, 82°, and 8° in the given triangle.

A right triangle is a type of triangle where one of the angles measures exactly 90 degrees. This angle is known as the right angle, and it is formed by the intersection of the two sides of the triangle that are perpendicular to each other. The other two angles of the right triangle are acute angles, meaning they measure less than 90 degrees.

The side opposite the right angle is called the hypotenuse, and it is always the longest side of the right triangle. The other two sides are called legs, and they can be of different lengths. This theorem is one of the most important and useful tools in geometry, and it allows us to solve many practical problems involving right triangles, such as finding the height of a building.

To learn more about Right triangle visit here:

brainly.com/question/26834721

#SPJ4

Consider a square whose side-length is one unit. Select any five points from inside this square. Prove that at least two of these points are within squareroot 2/2 units of each other.

Answers

The given square with a side length of one unit is known to contain five points. One must prove that at least two of these points are within square root 2/2 units of each other.

According to the Pigeonhole principle, "if n items are put into m containers, with n > m, then at least one container must contain more than one item."In this context, the square is the container, and the points inside it are the objects. If more than four points are picked, the theorem is true, and two points are nearer to each other than the square root of 2/2 units.

Let's place four points on the square's four corners. The distance between any two of these points is the square root of two units since the square's side length is 1.

Let's add another point to the mix. That point is either inside the square or outside it. Without loss of generality, let us assume that the point is inside the square. It must then be within the perimeter outlined by joining the square's corners to the point that was not a corner already.

The perimeter of the square described above is a square with a side length of square root 2 units.

Since we have five points in the square, at least two of them must be in the same smaller square, due to the pigeonhole principle. Without loss of generality, let's assume that two of the points are in the upper-left square. As a result, any points within this square are within the square root 2 units of any of the other four points. Hence, at least two points of the five selected are within the square root of 2/2 units of each other.

To know more about the "pigeonhole principle": https://brainly.com/question/13982786

#SPJ11

auditors compared opinions about treatment (very good/acceptable/poor) at four va hospitals (labeled a,b,c,d) among veterans aged 50 and above. what are the hypotheses for a chi-square test of independence on the data? select one:

Answers

The hypotheses for a chi-square test of independence on the data that auditors compared opinions about treatment (very good/acceptable/poor) at four VA hospitals (labeled a,b,c,d) among veterans aged 50 and above are:

Null hypothesis, H0: There is no association between the opinions about treatment of the VA hospitals and veterans aged 50 and above.

Alternative hypothesis, Ha: There is an association between the opinions about treatment of the VA hospitals and veterans aged 50 and above.

Hypothesis Testing is a type of statistical analysis in which you put your assumptions about a population parameter to the test. It is used to estimate the relationship between 2 statistical variables.

An analyst performs hypothesis testing on a statistical sample to present evidence of the plausibility of the null hypothesis. Measurements and analyses are conducted on a random sample of the population to test a theory. Analysts use a random population sample to test two hypotheses: the null and alternative hypotheses.

To learn more about null and alternative hypothesis refer :

https://brainly.com/question/27335001

#SPJ11

a garrison has provision for 30 days for certain men. if 2/3 of them do not attend the mess, then the food will last for? (b) 65 days (d) none of

Answers

A garrison has provision for 30 days for certain men. if 2/3 of them do not attend the mess, then the food will last for 45 days. So, the correct option is (a).

How to calculate

Given that the provision for certain men in the garrison is for 30 days. Also, given that 2/3 of them do not attend the mess, then we have to find the number of days the food will last.

The food will last longer if the number of people attending the mess is less because the same amount of food will have to be shared between fewer people. Therefore, the food will last for more than 30 days.

Let the total number of men be x, then the number of men attending the mess is (1/3)x

And the number of men not attending the mess is (2/3)x.

Therefore, the food will last for (30 × x) / (2/3)x = 45 days

Hence, the answer of the question is 46 days.

Your question is incomplete but most probably your full question was:

A garrison has provision for 30 days for certain men. if 2/3 of them do not attend the mess, then the food will last for?

(a) 45 days

(b) 65 days

(c) 50 days

(d) none of above

Learn more about fraction at

https://brainly.com/question/10354322

#SPJ11

you roll two six-sided dice at the same time. what is the probability the second dice landed on an even number given the first one landed on an odd number?

Answers

The probability that the second dice lands on an even number given that the first one landed on an odd number is 1/2 or 0.5.

Step-by-Step Explanation:Let A be the event that the first dice rolls an odd number, and let B be the event that the second dice rolls an even number. We want to find P(B|A), which is the probability of B given A. This can be found using the formula:P(B|A) = P(A and B) / P(A) We know that P(A) = 1/2, as half the numbers on a six-sided dice are odd.

To find P(A and B), we need to find the probability that both dice roll the desired numbers simultaneously. Since we know that the first dice rolled an odd number, only half of the numbers are possible for the second dice to be even. Therefore, P(A and B) = 1/2 x 1/2 = 1/4. So:P(B|A) = P(A and B) / P(A) = (1/4) / (1/2) = 1/2 or 0.5. Therefore, the probability that the second dice lands on an even number given that the first one landed on an odd number is 1/2 or 0.5.

See more about probability at: https://brainly.com/question/13604758

#SPJ11

Stacy rented a truck for one day there was a base fee of $16.95 and there was an additional charge of 93 cents for each model driven. stacy had to pay $143.43 when he returned the truck. for how many miles did she drive the truck​

Answers

Answer:

Stacy rented a truck for one day there was a base fee of $16.95 and there was an additional charge of 93 cents for each model driven. stacy had to pay $143.43 when he returned the truck. for how many miles did she drive the truck​

Step-by-step explanation:

Let's start by subtracting the base fee from the total cost:

$143.43 - $16.95 = $126.48

Now, we can divide the remaining cost by the cost per mile:

$126.48 ÷ $0.93/mile ≈ 136 miles

Therefore, Stacy drove the truck for approximately 136 miles.

Find the equation of the straight line passing through the point (0,2) which is perpendicular to the line y=1/4x+5

Answers

Answer:

y = -4x + 2

Step-by-step explanation:

you need to find the gradient first and in order to find it, you need to look at the equation of the line given

in the equation, it refers to y = mx + c and from there, the gradient is whatever the value of m is. So in this situation, m = 1/4

now that you've found your gradient, you need to get the gradient when it is perpendicular (as stated in the question) by using m¹ x m² = -1

m¹ represents the gradient of the line we have whereas m² represents the gradient of the line we want so you just have to substitute 1/4 into m¹

[tex] \frac{1}{4} \times {m}^{2} = - 1[/tex]

[tex] {m}^{2} = \frac{ - 1}{( \frac{1}{4} )} [/tex]

[tex] {m}^{2} = - 4[/tex]

now you need to find the c of the y = mx + c before you complete the equation

y = 2 (from the question)

x = 0 (from the question)

m = -4

(2) = (-4)(0) + c

2 = c

c = 2

and you just substitute everything except y into y = mx + c and you're done

y = -4x + 2

a country exports crayfish to overseas markets. the buyers are prepared to pay high prices when the crayfish arrive still alive. if x is the number of deaths per dozen crayfish, the probability distribution for x is given by: a. Find k. b. Over a long period, what is the mean number of deaths per dozen crayfish? c. Find σ, the standard deviation for the probability distribution.

Answers

a country exports crayfish to overseas markets. the buyers are prepared to pay high prices when the crayfish arrive still alive. if x is the number of deaths per dozen crayfish, the probability distribution for x is given by: a. Find k. b. Over a long period

A. To find k, you need to calculate the expected value of the random variable x, which is the number of deaths per dozen crayfish. This can be done by summing up the products of all the values of x multiplied by their respective probabilities. Thus,

                    k = ∑(xi * Pi)

                      = (1 * 0.3) + (2 * 0.3) + (3 * 0.2) + (4 * 0.2)

                      = 2.6

B. The mean number of deaths per dozen crayfish is given by the expected value of x, which is 2.6.

C. To find the standard deviation for the probability distribution, we need to calculate the variance of x. This can be done using the formula,

                 σ2 = ∑((xi - k)2 * Pi)

                      = (0 - 2.6)2 * 0.3 + (1 - 2.6)2 * 0.3 + (2 - 2.6)2 * 0.2 + (3 - 2.6)2 * 0.2

                      = 0.84

Therefore, the standard deviation for the probability distribution is σ = √0.84 = 0.92.

#SPJ11

Learn more about standard deviation for the probability distribution at: https://brainly.com/question/18804692

150,000 bonds with a coupon rate of 11 percent and a current price quote of 108; the bonds have 20 years to maturity. 320,000 zero coupon bonds with a price quote of 16 and 30 years until maturity. Both bonds have a par value of $1,000 and semiannual coupons

Answers

The total value of both bonds is $704,367,500.

Coupon payment = [tex]\frac{Coupon rate * Par value}{2}[/tex]

Coupon payment = [tex]\frac{11 * $1,000}{2}[/tex]

Coupon payment = $55

PV = [tex]55 * [1 - (1 + 0.04)^{^-40} ] / 0.04 + $1,000 / (1 + 0.04)^40[/tex]

[tex]PV = $1,026.45[/tex]

[tex]Total value = PV * Number of bonds * Par value\\Total value = $1,026.45 * 150,000 * $1,000\\Total value = $153,967,500[/tex]

[tex]PV = \frac{Price}{(1 + r)^n}[/tex]

[tex]PV = \frac{16}{(1 +0.03)^60}\\PV = $1.72[/tex]

[tex]Total value = PV * Number of bonds * Par value\\Total value = $1.72 * 320,000 * $1,000\\Total value = $550,400,000[/tex]

Therefore, the total value of both bonds is:

[tex]Total value = Value of coupon bonds + Value of zero coupon bonds\\Total value = $153,967,500 + $550,400,000\\Total value = $704,367,500[/tex]

A coupon rate is the annual interest rate paid by a bond or other fixed-income security to its bondholders or investors. It is typically expressed as a percentage of the bond's face value, also known as its par value. For example, if a bond has a face value of $1,000 and a coupon rate of 5%, the bond will pay $50 in interest each year to its bondholders. The coupon payments are usually made semi-annually or annually, depending on the terms of the bond.

The coupon rate is set when the bond is issued and remains fixed throughout the life of the bond unless the bond issuer chooses to call the bond or the bond defaults. Coupon rates are determined by a variety of factors, including market conditions, the creditworthiness of the issuer, and the length of the bond's maturity.

To learn more about Coupon rates visit here:

brainly.com/question/29765779

#SPJ4

Complete Question: -

The IPO Investment Bank has the following financing outstanding,

Debt: 150,000 bonds with a coupon rate of 11 percent and a current price quote of 108; the bonds have 20 years to maturity. 320,000 zero coupon bonds with a price quote of 16 and 30 years until maturity. Both bonds have a par value of $1,000 and semiannual coupons.

Preferred stock: 240,000 shares of 9 percent preferred stock with a current price of $67, and a par value of $100.

Common stock: 3,500,000 shares of common stock; the current price is $53, and the beta of the stock is.9.

Market: The corporate tax rate is 24 percent, the market risk premium is 8 percent, and the risk-free rate is 5 percent.

What is the WACC for the company? (Do not round intermediate calculations and enter your answer as a percent rounded to 2 decimal places, e.g., 32.16.)

Mr. And Mrs. Smith plan to roof the cabin on

2 consecutive days. Assuming that the chance of rain is

independent of the day, what is the probability that it

will rain both days?

A. 0. 04

B. 0. 08

C. 0. 16

D. 0. 20

E. 0. 40

Answers

From the given information provided, the probability that it will rain both days is 0.04 option A.

Since we are assuming that the chance of rain is independent of the day, we can use the multiplication rule of probability to find the probability that it will rain on both days.

Let's assume that the probability of rain on any given day is p. Then, the probability of no rain on that day is 1-p.

Therefore, the probability that it will rain on both days is:

P(rain on both days) = P(rain on day 1) × P(rain on day 2)

= p × p

= p²

Since the problem does not give us a specific value of p, we cannot determine the exact probability of rain on both days. However, we can use one of the answer choices to estimate the probability of rain on both days.

Looking at the answer choices, the only choice that is a perfect square is 0.04. Therefore, we can assume that p² = 0.04, which means that p = 0.2.

So, if the probability of rain on any given day is 0.2, then the probability of rain on both days is:

P(rain on both days) = p²

= 0.2²

= 0.04

Therefore, the answer is A. 0.04.

Learn more about probability here: brainly.com/question/24756209

#SPJ4

a blackboard of sides 5 M 30 cm and 3m 20 CM has to be painted find the cost of the rate of rs 15 per m².​

Answers

Answer: The cost of painting is Rs.254.4

Step-by-step explanation:

let  l and b be the sides

here l= 5m 30cm=5.30m

        b=3m 20 cm=3.20m    

Area of blackboard = l×b

                                          = 5.30×3.20

                                           =16.96m²

cost of painting per m² = 15 rscost of painting per 8.5m² = 16.96 × 15 =254.4 rs

                                                     

             

We are given that, the measures of sides of blackboard are 5 m 30 cm and 3m 20 cm.

__________________________________________

Length of the Blackboard

[tex] \bf \implies5 m + 30 cm \\ [/tex]

[tex] \sf \implies 5 m + \dfrac{30}{100}m \\ [/tex]

[tex] \sf \implies 5 m + \dfrac{3\cancel{0}}{10\cancel{0}}m \\ [/tex]

[tex] \sf \implies 5 m + 0.3 m \\ [/tex]

[tex]\purple{ \bf \implies 5.3~ m } \\ [/tex]

Breadth of the Blackboard

[tex] \bf \implies3 m + 20 cm \\ [/tex]

[tex] \sf \implies 3 m + \dfrac{20}{100}m \\ [/tex]

[tex] \sf \implies 3 m + \dfrac{2\cancel{0}}{10\cancel{0}}m \\ [/tex]

[tex] \sf \implies 3 m + 0.2 m \\ [/tex]

[tex] \purple{\bf \implies 3. 2 ~m} \\ [/tex]

_______________________________________________

[tex] \pink{\frak{\implies Area _{(Blackboard) }= Length \times Breadth ~m^2}} \\ [/tex]

[tex] \sf \implies Area _{(Blackboard) } = 5.3 \times 3.2 ~m^2 \\ [/tex]

[tex] \sf \implies Area _{(Blackboard) } = 16.96 m^2 \\ [/tex]

Henceforth, the cost of the rate of rs 15 per m² will be -

[tex] \sf \implies 15 \times 16.96 \\ [/tex]

[tex] \pink{\sf \implies Rs ~254.4 } \\ [/tex]

Two similar solids have base areas of 47 cm² and 199 cm², as shown below.
The volume of the smaller solid is 350 cm³.
COMPLETION
50%
Calculate the volume of the larger solid correct to the nearest integer.
(4 marks)

Answers

Check the picture below.

so hmmm let's use the ratio for the areas to get the ratio of the sides, and from there, we'll get to the ratio of the volumes.

[tex]\stackrel{ \textit{Areas' ratio} }{\sqrt{\cfrac{s^2}{s^2}}}=\cfrac{s}{s}\implies \sqrt{\cfrac{47}{199}}=\cfrac{s}{s}\implies \cfrac{\sqrt{47}}{\sqrt{199}}=\cfrac{s}{s} \\\\[-0.35em] ~\dotfill[/tex]

[tex]\stackrel{ \textit{Volumes' ratio} }{\sqrt[3]{\cfrac{s^3}{s^3}}}=\cfrac{s}{s}\implies \stackrel{\textit{substituting from above}}{\sqrt[3]{\cfrac{s^3}{s^3}}=\cfrac{\sqrt{47}}{\sqrt{199}}}\implies \sqrt[3]{\cfrac{350}{V}}=\cfrac{\sqrt{47}}{\sqrt{199}} \\\\\\ \cfrac{350}{V}=\left( \cfrac{\sqrt{47}}{\sqrt{199}} \right)^3\implies \cfrac{350}{V}=\cfrac{\sqrt{47^3}}{\sqrt{199^3}}\implies (350)(\sqrt{199^3})=V\sqrt{47^3} \\\\\\ \cfrac{(350)(\sqrt{199^3})}{\sqrt{47^3}}=V\implies \boxed{3049\approx V}[/tex]

You construct three 88% confidence intervals as follows: A) A t-interval with 6 degrees of freedom. B) A t-interval with 2 degrees of freedom. C) A z-interval Assuming the mean and standard deviation are the same for all three intervals, write the three intervals (A, B, and C) in order, from narrowest to widest.

Answers

The order from narrowest to widest is: C) z-interval. A) t-interval with 6 degrees of freedom. B) t-interval with 2 degrees of freedom.

What is confidence interval?

In statistics, the likelihood that a population parameter will fall between a set of values for a certain percentage of the time is referred to as a confidence interval. Analysts frequently employ confidence ranges that include 95% or 99% of anticipated observations. So, it may be concluded that there is a 95% likelihood that the real value falls within that range if a point estimate of 10.00 with a 95% confidence interval of 9.50 - 10.50 is derived using a statistical model.

A confidence interval's breadth is influenced by the sample size and degree of confidence. Higher confidence levels often result in broader intervals, whereas bigger sample numbers typically result in narrower intervals.

In this instance, the three intervals have different degrees of freedom but the same 88% confidence level.

Hence, The order from narrowest to widest is: C) z-interval. A) t-interval with 6 degrees of freedom. B) t-interval with 2 degrees of freedom.

Learn more about confidence interval here:

https://brainly.com/question/29680703

#SPJ1

Find the compound interest and the total amount after eight years if the interest is compounded every two years.
Principal = ₹10,000
Rate of interest = 20%
Total amount = (find)
Total interest = (find)

Answers

After 8 years, the total amount is ₹38,416 and the compound interest is ₹28,416.

What is the total amount and compound interest earned on ₹10,000 invested at 20% interest compounded every 2 years for 8 years?

To find the compound interest and the total amounts after eight year with interest compounded every two years, we'll use the compound interest formula:

Total Amount (A) = P(1 + r/n)¹/²(nt)

Where:
P = Principal = ₹10,000
r = Rate of interest = 20% = 0.2
n = Number of times the interest is compounded in a year (every 2 years, so n = 1/2)
t = Time in years = 8 years

Convert the interest rate to a decimal by dividing by 100:
20% ÷ 100 = 0.2

A = ₹10,000x (1 + 0.2¹/²)(1/2 x 8)

Calculate the expression inside the parentheses:
1 + 0.2/(1/2) = 1.4

Calculate the exponent (1/2x 8):
1/2x 8 = 4

Calculate the total amount:
A = ₹10,000 x (1.4)^4
A = ₹10,000 x3.8416
A = ₹38,416

Step 6: Calculate the compound interest:
Total interest = Total amount - Principal
Total interest = ₹38,416 - ₹10,000
Total interest = ₹28,416

So, after eight years, the total amount is ₹38,416 and the compound interest is ₹28,416.

Learn more about:compound interest

brainly.com/question/14295570

#SPJ11

Kay borrows $8,750 from Chase Bank at a fixed annual interest rate of 6.2%,
where y is the number of years the loan remains unpaid. Kay waits 8 years
before beginning to repay the loan to Citibank.
Write an exponential function to represent the amount owed after t years.
O y = 8750(t) 6.2
Oy = 8750 (1.062)
Oy (t) 1.0628750

Answers

Answer:

Answer: $80,000(1+0.004)^60 + $519.17 [1-(+0.004)^60/0.004]

Step-by-step explanation:

please help with question 6

Answers

Answer:

a = -13b = 6f(x) = (2x -1)(x -2)(x +3)

Step-by-step explanation:

Given f(x) = 2x³ +x² +ax +b has a factor (x -2) and a remainder of 18 when divided by (x -1), you want to know a, b, and the factored form of f(x).

Remainder

If (x -2) is a factor, then the value of f(2) is zero:

  f(2) = 2·2³ +2² +2a +b = 0

  2a +b = -20 . . . . . . . subtract 20

If the remainder from division by (x +1) is 18, then f(-1) is 18:

  f(-1) = 2·(-1)³ +(-1)² +a·(-1) +b = 18

  -a +b = 19 . . . . . . . . . . add 1

Solve for a, b

Subtracting the second equation from the first gives ...

  (2a +b) -(-a +b) = (-20) -(19)

  3a = -39

  a = -13

  b = 19 +a = 6

The values of 'a' and 'b' are -13 and 6, respectively.

Factored form

We can find the quadratic factor using synthetic division, given one root is x=2. The tableau for that is ...

  [tex]\begin{array}{c|cccc}2&2&1&-13&6\\&&4&10&-6\\\cline{1-5}&2&5&-3&0\end{array}[/tex]

The remainder is 0, as expected, and the quadratic factor of f(x) is 2x² +5x -3. Now, we know f(x) = (x -2)(2x² +5x -3).

To factor the quadratic, we need to find factors of (2)(-3) = -6 that have a sum of 5. Those would be 6 and -1. This lets us factor the quadratic as ...

  2x² +5x -3 = (2x +6)(2x -1)/2 = (x +3)(2x -1)

The factored form of f(x) is ...

  f(x) = (2x -1)(x -2)(x +3)

Which explicit function defines this arithmetic sequence?


-351, -343, -335, -327, -319

Answers

The explicit function defines this arithmetic sequence:

    C.  f(n) = 8n − 359

Explicit Function:

An explicit function is a function expressed with arguments. For example, y = 4x – 7 is self-explanatory, where y is the dependent variable and depends on the independent variable x.

According to the Question:

The first element of the given arithmetic sequence is -351, and the tolerance is (-343 -(-351)) = 8. The tolerance is a multiplier of n in the explicit function.

We can identify the appropriate explicit function by finding the one that correctly describes the sequence. Evaluating each for n=1 is sufficient.

(A) f(1) = 8 -351 = -343

(B) f(1) = -8 -351 = -359

(C) f(1) = 8 -359 = -351

(D) f(1) = -8 +359 = 351

The sequence is defined by the explicit function :

                  f(n) = 8n -359

Learn more about Sequence:

https://brainly.com/question/30262438

#SPJ4

HELP PLEASE.
The options of answer to this question are:
A- 286 miles
B- 271 miles
C- 166 miles
D- 216 miles
PLEASE HELP

Answers

Answer: D - 216 miles

Explanation: The rest stop is the y-intercept in the graph because that is when he begins to travel home at a constant speed. Looking at the graph, the y-intercept falls right before 220 so the closest answer would be 216 miles.

Let f(x) = (x - 1)2,g(x) = e-2 , and h(x) = 1 In(1 2x) _ (a) Find the linearizations of f, g, and h at a = 0
Lf(x)=1-2x
Lg(x)=1-2x
Lh(x)=1-2x
Graph f, g, and h and their linear approximations. For which function is the linear approximation best? For which is it worst? Explain.
The linear approximation appears to be the best for the function (f, g, or h *h is not the answer) since it is closer to (f, g, or h) for a larger domain than it is to (f and g, g and h, f and h). The approximation looks worst for (f, g, or, h) since (f, g, or, h) moves away from L faster than (f and g, g and h, f and h) do.

Answers

We can see that the linear approximation appears to be the best for function g since it is closer to g for a larger domain than it is to f and h. The approximation looks worst for function f since f moves away from Lf faster than g and h do. Hence, the linear approximation is the worst for function f.

Given that f(x) = (x - 1)^2, g(x) = e^(-2), and h(x) = ln(1+2x) - 2x.
(a) Find the linearizations of f, g, and h at a = 0.
The linearization of f at a = 0 is given by
Lf(x) = f(0) + f'(0)(x - 0)
     = (0 - 1)^2 + 2(0 - 1)(x - 0)
     = -x + 1
     The linearization of g at a = 0 is given by
Lg(x) = g(0) + g'(0)(x - 0)
     = e^(-2) + 0(x - 0)
     = e^(-2)
     The linearization of h at a = 0 is given by
Lh(x) = h(0) + h'(0)(x - 0)
     = (ln(1 + 2(0)) - 2(0)) + [(1/(1 + 2(0)))2 - 2](x - 0)
     = -2x
Graph f, g, and h and their linear approximations. For which function is the linear approximation best? For which is it worst? Explain.
The graphs of f(x) = (x - 1)^2, g(x) = e^(-2), h(x) = ln(1+2x) - 2x, and their respective linear approximations Lf(x) = -x + 1, Lg(x) = e^(-2), and Lh(x) = -2x.

For more questions on linear approximation

https://brainly.com/question/2254269

#SPJ11

Help!
I need your help.

Answers

a) 5x - 8y = -13 (x3)
2x - 3y = -4 (x8)

15x - 24y = -39 -
16x - 24y = -32
-x / = -7
x = -7/-1
x=7

Solve 2x-3y=-4 by using x=7
2x - 3y = -4
2(7) - 3y = -4
14 - 3y = -4
-3y = -4 -14
-3y = -18
y = -18/-3
y = 6

Therefore x = 7 and y = 6

PLEASE HELP MARKING BRAINLEIST JUST ANSWER ASAP

Answers

Answer: it’s B i think Explanation: it has a greater figure

Answer:

The pink triangle

Step-by-step explanation:

IMPORTANT NOTE: Make sure all the units are the same and consistent

Perimeter of a figure = Total length of the outer boundary

Shape of each figure in this question = Isosceles Triangle


Perimeter of triangle = Sum of all three sides


Perimeter of pink triangle = 36m + 36m + 20m

                                             = 92m

Perimeter of green triangle = 25m + 25m + 35m

                                             = 85m


∴Comparing the two values calculated above, it can be observed that the pink triangle has a greater perimeter

Fractions questions need help!

Answers

The answer to this question is 150 adults. This is calculated by subtracting the number of boys and girls from the total number of people in the museum, 250.

What is subtracting?

Subtracting is a mathematical operation that involves the removal of one number or quantity from another. Subtracting can be done by either counting down from the larger number or counting up from the smaller number until the two numbers meet.

2/5 of 250 people is equal to 100 girls. 3/10 of 250 people is equal to 75 boys. When these two numbers are subtracted from the total number of people in the museum, 250, the answer is 150 adults.

To work out the number of adults in the museum, it is important to first identify the fractions and convert them into decimals. For example, to convert 2/5 into a decimal, 2 is divided by 5, which gives an answer of 0.4. This process should be repeated for the other fractions given in this problem.

Once the fractions are converted into decimals, the next step is to multiply the decimals by the total number of people in the museum, 250. For example, 0.4 multiplied by 250 is equal to 100 girls.

Finally, the numbers of boys and girls should be subtracted from the total number of people in the museum, 250. This gives an answer of 150 adults.

For more questions related to fractions

https://brainly.com/question/20712359

#SPJ1

By subtracting the number of boys and girls from the total number of people in the museum, we get the number of adults that is 75.

What is subtracting?

Subtracting is a mathematical operation that involves the removal of one number or quantity from another. Subtracting can be done by either counting down from the larger number or counting up from the smaller number until the two numbers meet.

2/5 of 250 people = 100 girls.

3/10 of 250 people =75 boys.

When these two numbers are subtracted from the total number of people in the museum, that is

250-(100+75)= 75 adults

Thus, the number of adults among the 250 people in a museum are 75.

For more questions related to number

https://brainly.com/question/28467694

#SPJ1

You purchased 40 shares for $3.95/sh. If you sold the shares for a total of $200. Did you net a profit or a loss?

Answers

Answer: profit

Step-by-step explanation:

3.95 (price of one share) multiplied by 40 (the amount of shares bought) would have cost $158. so selling all for $200 would be a $42 profit

Other Questions
identify managerial levers that reduce a lot size and cycle inventory in a supply chain without increasing cost? Which of the following might lead to an increase in the equilibrium price of jelly and a decrease in the equilibrium quantity of jelly sold? a. an increase in the price of peanut better, a complement to jelly b. an increase in the price of Marshmallow Fluff, a substitute for jelly c. an increase in the price of grapes, an input into jelly d. an increase in consumers incomes, as long as jelly is a normal good in late april, the acme construction co. submitted a $1,200,000 progress billing on a construction contract. on may 2, the bill was approved for payment, subject to a ten percent retention, as provided by the contract. construction expenditures should be debited when: how does vegetation in wetlands reduce atmospheric carbon 5. Paul practiced playing the piafor hour on Friday. He practfor the same amount of timeon Saturday. How long did Ppractice on Saturday? Answer gets 90 points academic arguments cover a wide range of writing, but its harllmarks are an appeal to reason and a reliance on research Find the magnitude and direction of the vector . Round angles to the nearest degree and other values to the nearest tenth. 7.3; 737.3; 747.4; 737.4; 74 how do organisms obtain and use the matter and energy they need to live and grow? 7. Complete the comparison: 17>?O A. 18O B. 17O C. 39O D. 1O Mark for review will be highligh if an entity fails to disclose information in the financial statements or footnotes as required by gaap, the audit opinion options are In national income accounting, the consumption category of expenditures includes purchases of:a. both new and used consumer goods.b. automobiles for personal use, but not houses.c. consumer non-durable goods and services but not consumer durable goods.d. consumer durable and non-durable goods but not services. what is unique about the way greek monumental sacred architecture, like the parthenon, was used? Im not adverse to running, but 26. 2 miles is way to far for me after successfully isolating solid copper in part b of this experiment, bernice is wondering if there are other acids that could be used in place of the acids available in part b of this experiment. which of the following acids could be used instead of the provided acids (h2so4 and h3po4) to isolate solid copper in part b of this experiment? select all that applyo. HBro. HNO3o. H2So. H2CO3 developing countries with low saving rates and poor levels of health and education are likely to experience part 2 a. easy access to financial backing from banks. b. rapid growth in household incomes. c. low rates of growth in real gdp per capita. d. high levels of foreign direct investment. Find the equation of the straight line passing through the point (3,5) which is perpendicular to the line y=3x+2 Factor the polynomial completely:78 - 148 - 560s keisha has typed a professional email message to a buiness assciate. the next step for keisha to take is a manager who constantly emphasizes punctuality to subordinates arrives late to meetings. the nonverbal message will be more strongly believed by the subordinates. true or false Normally, the lungs function in a fairly high state of compliance. Which of the following could cause lung compliance to be abnormally high or low?atelectasispulmonary fibrosisemphysemaAll of the above are correct.